LSAT and Law School Admissions Forum

Get expert LSAT preparation and law school admissions advice from PowerScore Test Preparation.

User avatar
 Dave Killoran
PowerScore Staff
  • PowerScore Staff
  • Posts: 5853
  • Joined: Mar 25, 2011
|
#43585
Complete Question Explanation
(The complete setup for this game can be found here: lsat/viewtopic.php?t=8007)

The correct answer choice is (D)

From the first rule, when Y is selected, G cannot be selected, and therefore answer choice (A) is incorrect. As discussed in the game setup, when Y is selected, J cannot be selected, and thus answer choices (B), (C), and (E) can be eliminated. Answer choice (D) is thereby correct.
 scyq6@sina.com
  • Posts: 7
  • Joined: Jul 15, 2016
|
#27332
Hello,

I have trouble understanding why answer choice D and E are wrong. Or could you explain why B is the only group possible for this setting?

I suppose I get all of the information from the stimulus.

Thank you!
 Adam Tyson
PowerScore Staff
  • PowerScore Staff
  • Posts: 5153
  • Joined: Apr 14, 2011
|
#27334
Start by telling us about your setup, your diagram, and let's go from there. If you can show us how you diagrammed the rules and describe your base and any inferences you made, that will give us some idea of where you might have gone astray or misunderstood something. Better for us to focus on that than on just walking you through the whole setup.
 ValVal
  • Posts: 8
  • Joined: Jul 05, 2017
|
#36861
Hi, guys!

I also have a question about #16 in this game.
My set up is the following:
Y :dblline: G
H :dblline: G
H :arrow: K :arrow: X
J :arrow: W

So in Question 16, I got stuck between 2 answer choices: (A) and (B)
My reasoning is this:
If the hobbyist selects (A), which is W and X, he technically cannot select Y, because the plant group is full, nevertheless, the setup itself, as I see it, allows the possibility of choosing Y and thus violating the game scenario.

Almost identical reasoning behind answer choice (B): if he choses W and Y, he may or may not be prompted to chose X. So this option can equally violate or not to the game scenario.

So as you see, those answer choices both look pretty good to me.

Could you clarify this for me?

Thank you!
 Adam Tyson
PowerScore Staff
  • PowerScore Staff
  • Posts: 5153
  • Joined: Apr 14, 2011
|
#36882
Hey there, ValVal! It looks to me like you might be mixing up two questions here. Question 16 is a local question about what happens when the hobbyist selects Y, and we want to know which fish combination that could work with. The answer choices you are analyzing are the answer to question 17, which is a global question about possible plant combos. I'm thinking that you just got a little fatigued, maybe, and your eye just slid down to the wrong answer choices after you read the question? I know I've done that a time or three! By the way, your diagram looks picture perfect to me - good work!

Assuming for the moment that we are looking at the answer to question 16 now, we can quickly eliminate answer A because it has G in it, and Y and G will never get along. The issue for the remaining answer must be that three of them will cause us to violate a rule about the plants, either selecting too many, too few, or some that cannot get along with others.

Answer B causes a problem of too many plants - K requires X, as you diagrammed, and J requires W, also in your diagram. Those two plus Y make 3 plants - too many! It's out.

Answer C has a different problem - H requires K, which is missing from the list of plants.

Answer D is the right answer - plants H, K and L mean that X gets pulled in, and that gets along just fine with Y. Winner!

Answer E goes back to the earlier problem we had in answer B - J brings in W and K brings in X, and with Y that's too many.

Now, in case you meant to ask about question 17, I'll start another thread for that one!

Get the most out of your LSAT Prep Plus subscription.

Analyze and track your performance with our Testing and Analytics Package.